Jump to content

Lnmn179's Content

There have been 63 items by Lnmn179 (Search limited from 20-05-2020)



Sort by                Order  

#426858 $f(\frac{x-3}{x+1})+f(\frac{3+x}...

Posted by Lnmn179 on 13-06-2013 - 17:58 in Phương trình hàm

Bài 1: Ở đây . Đây là đề thi quốc gia Hàn Quốc hồi đó ( quên năm :D )

 

Bài 2 : Đặt $a=30f(0)$, thay $y=0$ ta có : $f(4x+a)=19x+2010$ => $f(x)$

 

Hai bài đa thức để suy nghĩ đã  :closedeyes: .

xin lỗi đã làm phiền, bạn có thể giải rõ bài 2 được không ? thông cảm, tại mình vốn dốt pt hàm  :( 




#426810 $f(\frac{x-3}{x+1})+f(\frac{3+x}...

Posted by Lnmn179 on 13-06-2013 - 15:52 in Phương trình hàm

1. Tìm hàm f thoả mãn $f(\frac{x-3}{x+1})+f(\frac{3+x}{1-x})=x$ với $x\neq 1,-1$

2. Tìm hàm f sao cho: $f(30f(y)+4x)=19x+75y+2010$ với $x,y\in R$

3. Tìm các đa thức với hệ số thực thoả mãn:

a) $(x-1)^{2}f(x)=(x-3)^{2}f(x+2)$ với thuộc R

b) $P(x^{2})+x(3P(x)+P(-x))=P(x)^{2}+2x^{2}$ với x thuộc R




#426648 $\frac{9x^{3}}{(\sqrt{3x+1}...

Posted by Lnmn179 on 13-06-2013 - 08:21 in Phương trình - hệ phương trình - bất phương trình

Bài 1: $\frac{9x^{3}}{(\sqrt{3x+1}-1)^{2}} = 6x^{2}-13x+8$

 

Bài 2: $\sqrt{2-x^{2}}+\sqrt{2-\frac{1}{x^{2}}}=4-(x+\frac{1}{x})$




#426130 Giải HPT $\left\{\begin{matrix} x^{3...

Posted by Lnmn179 on 11-06-2013 - 17:15 in Phương trình - hệ phương trình - bất phương trình

$\left\{\begin{matrix} x^{3}+y^{2}=(x-y)(xy-1)\\ x^{3}-x^{2}+y+1=xy(x-y+1)\end{matrix}\right.$

 




#416010 [HELP] AB // EF

Posted by Lnmn179 on 02-05-2013 - 16:59 in Hình học

Mọi người giải hộ em với! Phần b vs c e chưa làm đc. :(

e tks trước nha    :ukliam2:   :lol:

 

           Cho nửa đường tròn (O) đường kính AB. Điểm M thuộc nửa đường tròn, điểm C thuộc đoạn OA. Trên nửa mặt phẳng bờ là đường thẳng AB chứa điểm M vẽ tiếp tuyến Ax, By. Đường thẳng qua M vuông góc với MC cắt Ax, By lần lượt tại P và Q; AM cắt CP tại E, BM cắt CQ tại F.

 

a. CM tứ giác APMC nội tiếp đường tròn

b. CM $\angle PCQ=90^{\circ}$

c. CM AB//EF

 

:wub:   :wub:   :wub:

b) chứng minh tương tự ý a ta có tứ giác MCBQ nội tiếp.

=> $\angle PCM = \angle PAM$ và $\angle MCQ = \angle MBQ = \angle MAB$

=> $\angle PCQ = \angle PCM + \angle MCQ =\angle PAM + \angle MAB = \angle PAB = 90^{\circ}$

c) ta có $\angle PCQ = 90^{\circ} = \angle EMF$

=> EMFC là tứ giác nội tiếp

=> $\angle EFC = \angle EMC$\angle EFC = \angle QCB$$           

đồng thời $\angle EMC = \angle QMB$ (cùng bằng $90^{\circ} - \angle CMB$ )

và $\angle QMB = \angle QCB$ ( do tứ giác MQBC nội tiếp)

=> $\angle EFC = \angle QCB$

=> EF // AB (đpcm)




#409101 $\sum \frac{a}{\sqrt{a^{2}+...

Posted by Lnmn179 on 30-03-2013 - 16:37 in Bất đẳng thức và cực trị

Cho a,b,c > 0. CMR:$$\frac{a}{\sqrt{a^{2}+bc}} + \frac{b}{\sqrt{b^{2}+ca}}+\frac{c}{\sqrt{c^{2}+ab}}\geq 1$$




#406006 $\sum \sqrt{\frac{a^{2}+b^{2...

Posted by Lnmn179 on 18-03-2013 - 13:54 in Bất đẳng thức và cực trị

Cho a,b,c là các số thực dương thoả mãn $ab + bc + ca \leq 3abc$
CMR:
$\sqrt{\frac{a^{2}+b^{2}}{a+b}}+\sqrt{\frac{b^{2}+c^{2}}{b+c}} +\sqrt{\frac{c^{2}+a^{2}}{c+a}} +3 \leq \sqrt{2} (\sqrt{a+b}+\sqrt{b+c}+\sqrt{c+a})$



#394196 Cho a+b+c+d = 4 và a,b,c,d là các số dương

Posted by Lnmn179 on 07-02-2013 - 08:12 in Bất đẳng thức và cực trị

Cho a+b+c+d = 4 và a,b,c,d là các số dương. CMR:

$(a^{2}+2)(b^{2}+2)(c^{2}+2)(d^{2}+2)\geq 81$




#393736 $\frac{x^{2}+y^{2}}{x-y}...

Posted by Lnmn179 on 06-02-2013 - 12:11 in Số học

Tìm các cặp số nguyên dương x, y thoả mãn $\frac{x^{2}+y^{2}}{x-y}$ là số nguyên và là ước của 2010



#392895 $\frac{x! + y!}{n!} = 3^{n...

Posted by Lnmn179 on 03-02-2013 - 19:02 in Số học

Tìm các bộ số tự nhiên x,y,n thoả mãn :

$\frac{x! + y!}{n!} = 3^{n}$

qui ước 0! =1



#391765 Cho $H,I,O$ thuộc tam giác, CMR: $2IO\geq IH$

Posted by Lnmn179 on 30-01-2013 - 17:55 in Hình học

$9IG^2 = p^2 + 5r^2 - 16Rr$


Xin lỗi, bạn có thể giải rõ cho mình chỗ này được không. Mình cảm ơn



#391735 $ab^{2}+bc^{2}+ca^{2} \leq 4$

Posted by Lnmn179 on 30-01-2013 - 16:49 in Bất đẳng thức và cực trị

Cho $a,b,c\geq 0$ và thoả mãn a+b+c = 3.
CMR $ab^{2}+bc^{2}+ca^{2} \leq 4$
Dấu bằng xảy ra khi nào ?



#388755 $3^{n}\equiv 1$ (mod $n^{3}$)

Posted by Lnmn179 on 21-01-2013 - 13:29 in Các dạng toán THPT khác

Với $n=3$ thì $3^3=27$ chia cho $3^3=27$ hết mà.
Với $n=4$ thì $3^4=81$ chia cho $4^3=64$ dư $17$


Mình không hiểu ý bạn lắm. :wacko:



#388748 $3^{n}\equiv 1$ (mod $n^{3}$)

Posted by Lnmn179 on 21-01-2013 - 13:08 in Các dạng toán THPT khác

Tìm các số tự nhiên n lớn hơn 1 thoả mãn :
a, $3^{n}\equiv 1$ (mod $n^{3}$)
b, $5^{n}\equiv 1$ (mod $n^{3}$)



#388741 CMR K,I,P thẳng hàng

Posted by Lnmn179 on 21-01-2013 - 12:44 in Hình học phẳng

Cho $\Delta ABC$ ngoại tiếp đường tròn (I) và nội tiếp đường tròn (O). AI, BI cắt (O) lần lượt ở D, E. Giả sử DE cắt AC, BC lần lượt ở F, G. Đường thẳng qua F và song song với AI cắt đường thẳng qua G và song song với BI ở P. K là giao điểm các tiếp tuyến tại A, B của đường tròn (O). CMR K,I,P thẳng hàng



#383583 CMR D,G,X thẳng hàng

Posted by Lnmn179 on 04-01-2013 - 13:30 in Hình học phẳng

Cho $\Delta ABC$ với các góc nhọn, nội tiếp trong (O). $B_{0}$ là trung điểm AC. $C_{0}$ là trung điểm AB. Kẻ $AD\perp BC$ ($D\in BC$ ). G là trọng tâm $\Delta ABC$. W là tâm đường tròn qua $B_{0}$ và $C_{0}$ và tiếp xúc với (O) ở X. CMR: D,G,X thẳng hàng.



#376649 $(n+1)^n-1 \vdots n^2$

Posted by Lnmn179 on 10-12-2012 - 20:57 in Các dạng toán THPT khác

1. Cho $n\in N$. CMR:
a) ${3^{2}}^{4n+1}+{2^{3}}^{4n+1}+5\vdots 22$
b) $(n+1)^n-1 \vdots n^2$

2.Cho $a\geq 3; n> 1$ và thoả mãn $a^{n}\equiv 1 (mod n)$ với n là số tự nhiên. Gọi p là ước nguyên tố nhỏ nhất của n.
CMR : $a-1\vdots p$

3. Tìm $n\in N$ sao cho:
$2^{3n+4}+3^{2n+1}\vdots 19$



#369023 $B = a^2 b + a^2 c + ab^2 + ac^2 + b^2 c + bc^2 $

Posted by Lnmn179 on 12-11-2012 - 20:25 in Các bài toán Đại số khác

Đặt $ p = a+b+c; q = ab+ac+bc; r = abc$. Biến đổi các biểu thức sau theo $ p,q,r$:\[A = (a+b)(b+c)(a+c) ; B=a^2b+a^2c+ab^2+ac^2+b^2 c+bc^2 \]

___

NLT: Chú ý $\LaTeX$ và cách đặt tiêu đề !



#367432 Chứng minh rằng : luôn tìm được $a$ $;$ $b$ tro...

Posted by Lnmn179 on 06-11-2012 - 12:41 in Các dạng toán THPT khác

Cho $502$ số tự nhiên khác $0$ và phân biệt. Chứng minh rằng : luôn tìm được $a$ $;$ $b$ trong $502$ số sao cho :
$(a+b)$ $\vdots$ $1000$ hoặc $(a-b)$ $\vdots$ $1000$



#367430 Cho 17 số nguyên dương phân biệt

Posted by Lnmn179 on 06-11-2012 - 12:37 in Tổ hợp - Xác suất và thống kê - Số phức

Cho 17 số nguyên dương phân biệt. CMR: có thể chọn ra 9 số trong 17 số sao cho tổng 9 số đó chia hết cho 9



#367428 $a+b, b+c, c+a$ là số vô tỉ

Posted by Lnmn179 on 06-11-2012 - 12:33 in Các bài toán Đại số khác

Cho $6$ số vô tỉ tuỳ ý. CMR: luôn chọn được ra $3 $ số $a,b,c$ sao cho $a+b, b+c, c+a$ đều là số vô tỉ



#366927 $MA^{4}+MB^{4}+MC^{4}=const$

Posted by Lnmn179 on 04-11-2012 - 08:45 in Hình học phẳng

+) Xét M thuộc cung nhỏ BC:
=> MA - MB = MC.
$MA^{4}+MB^{4}+MC^{4}$
= $MA^{4}+MB^{4}+\left ( MA-MB \right )^{4}$
= $2(MA^{4}+MB^{4})-2(2MA^{3}MB-3MA^{2}MB^{2}+2MB^{3}MA)$
= $2(MA^{4}+MB^{4}-2MA^{3}MB+3MA^{2}MB^{2}-2MB^{3}MA)$
= $2(MA^{4}+MB^{4}-2MA^{3}MB+2MA^{2}MB^{2}-2MB^{3}MA+MA^{2}MB^{2})$
= $2(MA^{2}+MB^{2}-MA\cdot MB)^{2}$ (1)
Lấy P thuộc MA sao cho PM = BM.
=> AP = CM
Kẻ $CH \perp AM (H\in AM)$
ta có : $AB^{2}=AC^{2}=AH^{2}+CH^{2}= AM^{2}+BM^{2}-MA\cdot MB$
thay vào (1) ta có $MA^{4}+MB^{4}+MC^{4}$ = $2 AB^{4}$ = const
+) Trường hợp M thuộc cung nhỏ AB, AC cm tương tự.



#364291 Tìm vị trí điểm M sao cho MH +MK+ ML max, min

Posted by Lnmn179 on 23-10-2012 - 20:53 in Hình học phẳng

t k hiểu đoạn tô màu


đơn giản thôi bạn. Vì $\overrightarrow{MH}$ và $\overrightarrow{IX}$ cùng phương và cùng hướng nên tích vô hướng của chúng bằng tích độ dài của chúng. ( vì cos$\left ( \overrightarrow{MH},\overrightarrow{IX} \right )$ =1 )
tương tự với hai tích còn lại.



#363941 Tìm vị trí điểm M sao cho MH +MK+ ML max, min

Posted by Lnmn179 on 22-10-2012 - 21:29 in Hình học phẳng

Gọi X, Y, Z lần lượt là giao điểm của (I) với BC, CA, AB. r là bán kính của (I). G là trọng tâm tam giác XYZ.
ta có :
MH+MK+ML
= $\frac{1}{r}(MH\cdot IX+MK\cdot IY+ML\cdot IZ )$
= $\frac{1}{r}(\overrightarrow{MH}\cdot \overrightarrow{IX}+\overrightarrow{MK}\cdot \overrightarrow{IY}+\overrightarrow{ML}\cdot \overrightarrow{IZ})$
= $\frac{1}{r}(\overrightarrow{MX}\cdot \overrightarrow{IX}+\overrightarrow{MY}\cdot \overrightarrow{IY}+\overrightarrow{MZ}\cdot \overrightarrow{IZ})$
= $\frac{1}{r}((\overrightarrow{MI}+\overrightarrow{IX})\overrightarrow{IX}+(\overrightarrow{MI}+\overrightarrow{IY})\overrightarrow{IY}+(\overrightarrow{MI}+\overrightarrow{IZ})\overrightarrow{IZ})$
= $\frac{1}{r}(\overrightarrow{MI}(\overrightarrow{IX}+\overrightarrow{IY}+\overrightarrow{IZ})+3r^{2})$
= $3r+\frac{-3}{r}\overrightarrow{IG}\cdot \overrightarrow{IM}$
=> MH+MK+ML max(min)
<=> $\overrightarrow{IM}\cdot \overrightarrow{IG}$ min(max)
<=> $\overrightarrow{IM}$ và $\overrightarrow{IG}$ ngược hướng ( cùng hướng )
<=> M trùng P hoặc M trùng Q.
với PQ là đường kính (I), $\overrightarrow{IP}$ cùng hướng $\overrightarrow{IG}$, $\overrightarrow{IQ}$ ngược hướng $\overrightarrow{IG}$



#350947 $x^{4}(y+z)+y^{4}(z+x)+z^{4}(x+y)\leq...

Posted by Lnmn179 on 30-08-2012 - 17:48 in Bất đẳng thức và cực trị

cho $x,y,z\geq 0$. CMR

$x^{4}(y+z)+y^{4}(z+x)+z^{4}(x+y)\leq \frac{1}{12}(x+y+z)^{5}$